Is Openess and closedness are preserved under uniformly continuous map? [on hold]











up vote
-3
down vote

favorite
1












Question: does uniformly continuous function maps open set to open set? and does uniformly continuous function maps closed set to closed set?



"I think they are false" but I couldn't able to find counter examples.plese help me..










share|cite|improve this question















put on hold as off-topic by Brahadeesh, user21820, Holo, TheSimpliFire, amWhy yesterday


This question appears to be off-topic. The users who voted to close gave this specific reason:


  • "This question is missing context or other details: Please improve the question by providing additional context, which ideally includes your thoughts on the problem and any attempts you have made to solve it. This information helps others identify where you have difficulties and helps them write answers appropriate to your experience level." – Brahadeesh, user21820, Holo, TheSimpliFire, amWhy

If this question can be reworded to fit the rules in the help center, please edit the question.













  • I don't know why negative votes? Isn't community is for those, who seeking help in Mathematics? Yes I am beiginner and doesn't able to find counter example. So what's wrong in it.
    – Akash Patalwanshi
    yesterday

















up vote
-3
down vote

favorite
1












Question: does uniformly continuous function maps open set to open set? and does uniformly continuous function maps closed set to closed set?



"I think they are false" but I couldn't able to find counter examples.plese help me..










share|cite|improve this question















put on hold as off-topic by Brahadeesh, user21820, Holo, TheSimpliFire, amWhy yesterday


This question appears to be off-topic. The users who voted to close gave this specific reason:


  • "This question is missing context or other details: Please improve the question by providing additional context, which ideally includes your thoughts on the problem and any attempts you have made to solve it. This information helps others identify where you have difficulties and helps them write answers appropriate to your experience level." – Brahadeesh, user21820, Holo, TheSimpliFire, amWhy

If this question can be reworded to fit the rules in the help center, please edit the question.













  • I don't know why negative votes? Isn't community is for those, who seeking help in Mathematics? Yes I am beiginner and doesn't able to find counter example. So what's wrong in it.
    – Akash Patalwanshi
    yesterday















up vote
-3
down vote

favorite
1









up vote
-3
down vote

favorite
1






1





Question: does uniformly continuous function maps open set to open set? and does uniformly continuous function maps closed set to closed set?



"I think they are false" but I couldn't able to find counter examples.plese help me..










share|cite|improve this question















Question: does uniformly continuous function maps open set to open set? and does uniformly continuous function maps closed set to closed set?



"I think they are false" but I couldn't able to find counter examples.plese help me..







uniform-continuity open-map closed-map






share|cite|improve this question















share|cite|improve this question













share|cite|improve this question




share|cite|improve this question








edited yesterday

























asked yesterday









Akash Patalwanshi

9011816




9011816




put on hold as off-topic by Brahadeesh, user21820, Holo, TheSimpliFire, amWhy yesterday


This question appears to be off-topic. The users who voted to close gave this specific reason:


  • "This question is missing context or other details: Please improve the question by providing additional context, which ideally includes your thoughts on the problem and any attempts you have made to solve it. This information helps others identify where you have difficulties and helps them write answers appropriate to your experience level." – Brahadeesh, user21820, Holo, TheSimpliFire, amWhy

If this question can be reworded to fit the rules in the help center, please edit the question.




put on hold as off-topic by Brahadeesh, user21820, Holo, TheSimpliFire, amWhy yesterday


This question appears to be off-topic. The users who voted to close gave this specific reason:


  • "This question is missing context or other details: Please improve the question by providing additional context, which ideally includes your thoughts on the problem and any attempts you have made to solve it. This information helps others identify where you have difficulties and helps them write answers appropriate to your experience level." – Brahadeesh, user21820, Holo, TheSimpliFire, amWhy

If this question can be reworded to fit the rules in the help center, please edit the question.












  • I don't know why negative votes? Isn't community is for those, who seeking help in Mathematics? Yes I am beiginner and doesn't able to find counter example. So what's wrong in it.
    – Akash Patalwanshi
    yesterday




















  • I don't know why negative votes? Isn't community is for those, who seeking help in Mathematics? Yes I am beiginner and doesn't able to find counter example. So what's wrong in it.
    – Akash Patalwanshi
    yesterday


















I don't know why negative votes? Isn't community is for those, who seeking help in Mathematics? Yes I am beiginner and doesn't able to find counter example. So what's wrong in it.
– Akash Patalwanshi
yesterday






I don't know why negative votes? Isn't community is for those, who seeking help in Mathematics? Yes I am beiginner and doesn't able to find counter example. So what's wrong in it.
– Akash Patalwanshi
yesterday












1 Answer
1






active

oldest

votes

















up vote
4
down vote



accepted










Try $f(x)=sin x$. This has range $[-1,1]$. Also consider the image of
a set like ${2npi+1/n:ninBbb N}$.






share|cite|improve this answer





















  • Thanks sir, I think , the set ${2nπ+ 1/n: nin mathbb{N}}$ is closed as limit point $0$ is not in set and its image will be set $f(S)={sin(1/n): nin mathbb{N}}$ which is not closed as $sin(0)=0$ is not in $f(S)$ ? Am I correct? Further what about Openess?
    – Akash Patalwanshi
    yesterday












  • Sir please elaborate
    – Akash Patalwanshi
    yesterday






  • 1




    @AkashPatalwanshi That's right. But although terse, my answer did address both openness and closedness. $ddotsmile$
    – Lord Shark the Unknown
    yesterday












  • Oh got it sir...$ddotsmile$ is you want to say, $f(mathbb{R})=[-1,1]$ and hence f doesn't maps open set to an open set?
    – Akash Patalwanshi
    yesterday








  • 1




    @AkashPatalwanshi That's correct.
    – Lord Shark the Unknown
    yesterday


















1 Answer
1






active

oldest

votes








1 Answer
1






active

oldest

votes









active

oldest

votes






active

oldest

votes








up vote
4
down vote



accepted










Try $f(x)=sin x$. This has range $[-1,1]$. Also consider the image of
a set like ${2npi+1/n:ninBbb N}$.






share|cite|improve this answer





















  • Thanks sir, I think , the set ${2nπ+ 1/n: nin mathbb{N}}$ is closed as limit point $0$ is not in set and its image will be set $f(S)={sin(1/n): nin mathbb{N}}$ which is not closed as $sin(0)=0$ is not in $f(S)$ ? Am I correct? Further what about Openess?
    – Akash Patalwanshi
    yesterday












  • Sir please elaborate
    – Akash Patalwanshi
    yesterday






  • 1




    @AkashPatalwanshi That's right. But although terse, my answer did address both openness and closedness. $ddotsmile$
    – Lord Shark the Unknown
    yesterday












  • Oh got it sir...$ddotsmile$ is you want to say, $f(mathbb{R})=[-1,1]$ and hence f doesn't maps open set to an open set?
    – Akash Patalwanshi
    yesterday








  • 1




    @AkashPatalwanshi That's correct.
    – Lord Shark the Unknown
    yesterday















up vote
4
down vote



accepted










Try $f(x)=sin x$. This has range $[-1,1]$. Also consider the image of
a set like ${2npi+1/n:ninBbb N}$.






share|cite|improve this answer





















  • Thanks sir, I think , the set ${2nπ+ 1/n: nin mathbb{N}}$ is closed as limit point $0$ is not in set and its image will be set $f(S)={sin(1/n): nin mathbb{N}}$ which is not closed as $sin(0)=0$ is not in $f(S)$ ? Am I correct? Further what about Openess?
    – Akash Patalwanshi
    yesterday












  • Sir please elaborate
    – Akash Patalwanshi
    yesterday






  • 1




    @AkashPatalwanshi That's right. But although terse, my answer did address both openness and closedness. $ddotsmile$
    – Lord Shark the Unknown
    yesterday












  • Oh got it sir...$ddotsmile$ is you want to say, $f(mathbb{R})=[-1,1]$ and hence f doesn't maps open set to an open set?
    – Akash Patalwanshi
    yesterday








  • 1




    @AkashPatalwanshi That's correct.
    – Lord Shark the Unknown
    yesterday













up vote
4
down vote



accepted







up vote
4
down vote



accepted






Try $f(x)=sin x$. This has range $[-1,1]$. Also consider the image of
a set like ${2npi+1/n:ninBbb N}$.






share|cite|improve this answer












Try $f(x)=sin x$. This has range $[-1,1]$. Also consider the image of
a set like ${2npi+1/n:ninBbb N}$.







share|cite|improve this answer












share|cite|improve this answer



share|cite|improve this answer










answered yesterday









Lord Shark the Unknown

96.9k958128




96.9k958128












  • Thanks sir, I think , the set ${2nπ+ 1/n: nin mathbb{N}}$ is closed as limit point $0$ is not in set and its image will be set $f(S)={sin(1/n): nin mathbb{N}}$ which is not closed as $sin(0)=0$ is not in $f(S)$ ? Am I correct? Further what about Openess?
    – Akash Patalwanshi
    yesterday












  • Sir please elaborate
    – Akash Patalwanshi
    yesterday






  • 1




    @AkashPatalwanshi That's right. But although terse, my answer did address both openness and closedness. $ddotsmile$
    – Lord Shark the Unknown
    yesterday












  • Oh got it sir...$ddotsmile$ is you want to say, $f(mathbb{R})=[-1,1]$ and hence f doesn't maps open set to an open set?
    – Akash Patalwanshi
    yesterday








  • 1




    @AkashPatalwanshi That's correct.
    – Lord Shark the Unknown
    yesterday


















  • Thanks sir, I think , the set ${2nπ+ 1/n: nin mathbb{N}}$ is closed as limit point $0$ is not in set and its image will be set $f(S)={sin(1/n): nin mathbb{N}}$ which is not closed as $sin(0)=0$ is not in $f(S)$ ? Am I correct? Further what about Openess?
    – Akash Patalwanshi
    yesterday












  • Sir please elaborate
    – Akash Patalwanshi
    yesterday






  • 1




    @AkashPatalwanshi That's right. But although terse, my answer did address both openness and closedness. $ddotsmile$
    – Lord Shark the Unknown
    yesterday












  • Oh got it sir...$ddotsmile$ is you want to say, $f(mathbb{R})=[-1,1]$ and hence f doesn't maps open set to an open set?
    – Akash Patalwanshi
    yesterday








  • 1




    @AkashPatalwanshi That's correct.
    – Lord Shark the Unknown
    yesterday
















Thanks sir, I think , the set ${2nπ+ 1/n: nin mathbb{N}}$ is closed as limit point $0$ is not in set and its image will be set $f(S)={sin(1/n): nin mathbb{N}}$ which is not closed as $sin(0)=0$ is not in $f(S)$ ? Am I correct? Further what about Openess?
– Akash Patalwanshi
yesterday






Thanks sir, I think , the set ${2nπ+ 1/n: nin mathbb{N}}$ is closed as limit point $0$ is not in set and its image will be set $f(S)={sin(1/n): nin mathbb{N}}$ which is not closed as $sin(0)=0$ is not in $f(S)$ ? Am I correct? Further what about Openess?
– Akash Patalwanshi
yesterday














Sir please elaborate
– Akash Patalwanshi
yesterday




Sir please elaborate
– Akash Patalwanshi
yesterday




1




1




@AkashPatalwanshi That's right. But although terse, my answer did address both openness and closedness. $ddotsmile$
– Lord Shark the Unknown
yesterday






@AkashPatalwanshi That's right. But although terse, my answer did address both openness and closedness. $ddotsmile$
– Lord Shark the Unknown
yesterday














Oh got it sir...$ddotsmile$ is you want to say, $f(mathbb{R})=[-1,1]$ and hence f doesn't maps open set to an open set?
– Akash Patalwanshi
yesterday






Oh got it sir...$ddotsmile$ is you want to say, $f(mathbb{R})=[-1,1]$ and hence f doesn't maps open set to an open set?
– Akash Patalwanshi
yesterday






1




1




@AkashPatalwanshi That's correct.
– Lord Shark the Unknown
yesterday




@AkashPatalwanshi That's correct.
– Lord Shark the Unknown
yesterday



Popular posts from this blog

flock() on closed filehandle LOCK_FILE at /usr/bin/apt-mirror

Mangá

 ⁒  ․,‪⁊‑⁙ ⁖, ⁇‒※‌, †,⁖‗‌⁝    ‾‸⁘,‖⁔⁣,⁂‾
”‑,‥–,‬ ,⁀‹⁋‴⁑ ‒ ,‴⁋”‼ ⁨,‷⁔„ ‰′,‐‚ ‥‡‎“‷⁃⁨⁅⁣,⁔
⁇‘⁔⁡⁏⁌⁡‿‶‏⁨ ⁣⁕⁖⁨⁩⁥‽⁀  ‴‬⁜‟ ⁃‣‧⁕‮ …‍⁨‴ ⁩,⁚⁖‫ ,‵ ⁀,‮⁝‣‣ ⁑  ⁂– ․, ‾‽ ‏⁁“⁗‸ ‾… ‹‡⁌⁎‸‘ ‡⁏⁌‪ ‵⁛ ‎⁨ ―⁦⁤⁄⁕